1
$\begingroup$

If $f \in M_k(N, \chi)$ is a normalized integer weight Hecke eigenform for all $T_p$, is there a way to obtain $g \in M_k \left(4N, \chi \left( \frac{-4}{\cdot} \right) \right)$ via the use of operators in the theory of modular forms?

$\endgroup$
5
  • 2
    $\begingroup$ Do you mean $g \in M_k(4N,\chi(\frac{-4}{\cdot}))$ rather than $f \in M_k(N,\chi(\frac{-4}{\cdot}))$? Otherwise this is obviously impossible. $\endgroup$ Commented Mar 21 at 22:01
  • $\begingroup$ Yes, I have edited the question. Sorry for the confusion. $\endgroup$ Commented Mar 21 at 22:06
  • $\begingroup$ Just to be sure: you want forms with the same weight? $\endgroup$ Commented Mar 22 at 9:15
  • $\begingroup$ Yes, the forms should have same weight $k \in \mathbb{Z}$. $\endgroup$ Commented Mar 22 at 16:18
  • 3
    $\begingroup$ Impossible since $\chi(-1)=(-1)^k$ and muliplying by $(-4/.)$ changes parity. $\endgroup$ Commented Mar 22 at 17:05

0

You must log in to answer this question.

Start asking to get answers

Find the answer to your question by asking.

Ask question

Explore related questions

See similar questions with these tags.